Đến nội dung

Hình ảnh

Bất đẳng thức qua các kì thi Olimpic

- - - - -

  • Please log in to reply
Chủ đề này có 53 trả lời

#1
alex_hoang

alex_hoang

    Thượng úy

  • Hiệp sỹ
  • 1152 Bài viết

Bất đẳng thức qua các kì thi Olimpic


Trong chủ đề này mình và các bạn cùng trao đổi về các BĐT olympic để đưa ra những lời giải cho nó và những MỞ RỘNG
ví dụ như trong kì thi IMO năm 2008 bài bất đẳng thc khá hay và có thẻ tổng quát như sau
x,y,z,m là các số thực thỏa mãn xyz=1 CMR
${\left( {\dfrac{{x + m}}{{x - 1}}} \right)^2} + {\left( {\dfrac{{y + m}}{{y - 1}}} \right)^2}+ {\left( {\dfrac{{z + m}}{{z - 1}}} \right)^2} \ge 1.$
Trước khi nó bị xóa tại topic về BĐT mình đã kịp có lời giải như sau
Dặt $a = \dfrac{{x + m}}{{x - 1}},b = \dfrac{{y + m}}{{y - 1}},z = \dfrac{{z + m}}{{z - 1}}$
$ \Rightarrow x = \dfrac{{m + a}}{{a - 1}},y = \dfrac{{m + b}}{{b - 1}},z = \dfrac{{m + c}}{{c - 1}}$
Do $xyz=1$ nên $(m+a)(m+b)(m+a)=(a-1)(b-1)(c-1)$
khai triển ta được $\[({m^2} - 1)(a + b + c) + (m + 1)(ab + bc + ca) + ({m^3} + 1) = 0\]$
Với $m=-1$ hiển nhiên BĐT đung
Với $m \neq -1$ ta có $(m - 1)(a + b + c) + (ab + bc + ca) + ({m^2} - m + 1) = 0$
Vậy ${a^2} + {b^2} + {c^2} = {a^2} + {b^2} + {c^2} + 2((m - 1)(a + b + c) + (ab + bc + ca) + {m^2} - m + 1) $
$= {(a + b + c)^2} + 2(m - 1)(a + b + c) + {(m - 1)^2} + {m^2} + 1 $
$= {(a + b + c + m - 1)^2} + {m^2} + 1 \ge 1$
Ta có đpcm
Mở đầu chủ đề nè
Bài 1:(Croatia TST 2011)cho các số thực dương a,b,c thỏa mãn $a+b+c=3$.CMR
$\dfrac{{{a^2}}}{{a + {b^2}}} + \dfrac{{{b^2}}}{{b + {c^2}}} + \dfrac{{{c^2}}}{{c + {a^2}}} \ge \dfrac{3}{2}$
Bài 2: Cho các số thực a,b,c,d khác 0 thỏa $a+b+c+d=0$ Dặt ${S_1} = ab + bc + cd;{S_2} = ac + ad + db$
CMR $S = \alpha {S_1} + \beta {S_2} \le 0$
Đây là bài tổng quát BĐT trong kì thi Croatia 1996

Bài viết đã được chỉnh sửa nội dung bởi alex_hoang: 30-08-2011 - 20:49
Gõ tiếng Việt đầy đủ dấu

alex_hoang


HẸN NGÀY TRỞ LẠI VMF THÂN MẾN

http://www.scribd.co...oi-Ban-Cung-The

#2
truclamyentu

truclamyentu

    Sĩ quan

  • Thành viên
  • 333 Bài viết

Bài 1:(Croatia TST 2011)cho các số thực dương a,b,c thỏa mãn $a+b+c=3$.CMR
$\[\dfrac{{{a^2}}}{{a + {b^2}}} + \dfrac{{{b^2}}}{{b + {c^2}}} + \dfrac{{{c^2}}}{{c + {a^2}}} \ge \dfrac{3}{2}\]$



Bài 1:

$\begin{array}{l}\sum {\dfrac{{{a^2}}}{{a + {b^2}}}} = \sum {\dfrac{{a(a + {b^2}) - a{b^2}}}{{a + {b^2}}}} = \sum {a - \sum {\dfrac{{a{b^2}}}{{a + {b^2}}}} } \\\\\ge 3 - \sum {\dfrac{{a{b^2}}}{{2b\sqrt a }}} = 3 - \dfrac{1}{2}\sum {b\sqrt a } = 3 - \dfrac{1}{2}\sum {\sqrt {ba} \sqrt b } \ge 3 - \dfrac{1}{4}\sum {(ab + b)} \\\\ = 3 - \dfrac{1}{4}\sum b - \dfrac{1}{4}\sum {ab \ge \dfrac{9}{4}} - \dfrac{1}{4}.\dfrac{1}{3}({\sum {a)} ^2} = \dfrac{3}{2}\end{array}$

Bài viết đã được chỉnh sửa nội dung bởi dark templar: 28-05-2011 - 18:19


#3
alex_hoang

alex_hoang

    Thượng úy

  • Hiệp sỹ
  • 1152 Bài viết

1//

$\begin{array}{l}\sum {\dfrac{{{a^2}}}{{a + {b^2}}}} = \sum {\dfrac{{a(a + {b^2}) - a{b^2}}}{{a + {b^2}}}} = \sum {a - \sum {\dfrac{{a{b^2}}}{{a + {b^2}}}} } \\\\\ge 3 - \sum {\dfrac{{a{b^2}}}{{2b\sqrt a }}} = 3 - \dfrac{1}{2}\sum {b\sqrt a } = 3 - \dfrac{1}{2}\sum {\sqrt {ba} \sqrt b } \ge 3 - \dfrac{1}{4}\sum {(ab + b)} \\\\ = 3 - \dfrac{1}{4}\sum b - \dfrac{1}{4}\sum {ab \ge \dfrac{9}{4}} - \dfrac{1}{4}.\dfrac{1}{3}({\sum {a)} ^2} = \dfrac{3}{2}\end{array}$

Bài 1 mình giải thế này
Áp dụng BĐT Cauchy sch­warz ta có
$\dfrac{{{a^2}}}{{a + {b^2}}} + \dfrac{{{b^2}}}{{b + {c^2}}} + \dfrac{{{c^2}}}{{c + {a^2}}} \ge \dfrac{{{{({a^2} + {b^2} + {c^2})}^2}}}{{{a^3} + {b^3} + {c^3} + {{(ab)}^2} + {{(bc)}^2} + {{(ca)}^2}}}$
$2{({a^2} + {b^2} + {c^2})^2} \ge 3({a^3} + {b^3} + {c^3} + {(ab)^2} + {(bc)^2} + {(ca)^2}) $
$= (a + b + c)[{a^3} + {b^3} + {c^3}) + 3({(ab)^2} + {(bc)^2} + {(ca)^2})]$
tù đay áp dụng BDT vasile citoaje
${({a^2} + {b^2} + {c^2})^2} \ge 3(a{b^3} + b{c^3} + c{a^3})$
Bài toán trên không khó nhưng nó là một hệ quả đẹp của BDTvasile citoaje có 2 bài toán nữa cũng tương tự về hình thức cũng như ppcm là
*cho các số thực dương a,b,c có tổng là 3 cm
$\dfrac{a}{{b + {c^2}}} + \dfrac{b}{{c + {a^2}}} + \dfrac{c}{{a + {b^2}}} \ge 3$
* cho các số thực dương a,b,c có tổng là 1 cm
$\dfrac{a}{{\sqrt {a + {b^2}} }} + \dfrac{b}{{\sqrt {b + {c^2}} }} + \dfrac{c}{{\sqrt {c + {a^2}} }} \le \dfrac{3}{2}$


Bài 3:(Canada 2008) cho các số thực dương a,b,c có tổng là 1.CM
$\dfrac{{a - bc}}{{a + bc}} + \dfrac{{b - ca}}{{b + ca}} + \dfrac{{c - ab}}{{c + ab}} \le \dfrac{3}{2}$

Bài viết đã được chỉnh sửa nội dung bởi dark templar: 28-05-2011 - 18:26

alex_hoang


HẸN NGÀY TRỞ LẠI VMF THÂN MẾN

http://www.scribd.co...oi-Ban-Cung-The

#4
truclamyentu

truclamyentu

    Sĩ quan

  • Thành viên
  • 333 Bài viết

Bài 3(Canada 2008) cho các số thực dương a,b,c có tổng là 1.CM
$\[\dfrac{{a - bc}}{{a + bc}} + \dfrac{{b - ca}}{{b + ca}} + \dfrac{{c - ab}}{{c + ab}} \le \dfrac{3}{2}\]$


$\sum {\dfrac{{a - bc}}{{a + bc}}} \le \dfrac{3}{2} \Leftrightarrow \sum {\dfrac{{bc}}{{a + bc}}} \ge \dfrac{3}{4} \Leftrightarrow \sum {\dfrac{{bc}}{{(a + b)(a + c)}}} \ge \dfrac{3}{4}\\\\ \Leftrightarrow 4\sum {bc(b + c) \ge } 3(a + b)(b + c)(c + a)\\\\ \Leftrightarrow \sum {bc(b + c) \ge } 6abc$

bất đẳng thức này chứng minh dễ dàng bằng côsi . vậy BĐT cần CM cũng đúng

Bài viết đã được chỉnh sửa nội dung bởi truclamyentu: 28-05-2011 - 10:22


#5
alex_hoang

alex_hoang

    Thượng úy

  • Hiệp sỹ
  • 1152 Bài viết
[quote name='truclamyentu' date='May 28 2011, 10:19 AM' post='262415']
Bài 4 (chọn học sinh giỏi tỉnh Nghệ An 2010) cho các số thực dương a,b,c tỏa mãn $\[9({a^4} + {b^4} + {c^4}) - 25({a^2} + {b^2} + {c^2}) + 48 = 0\]$.Tìm GTNN của biểu thức
$\[\dfrac{{{a^2}}}{{b + 2c}} + \dfrac{{{b^2}}}{{c + 2c}} + \dfrac{{{c^2}}}{{a + 2b}}\]$
alex_hoang


HẸN NGÀY TRỞ LẠI VMF THÂN MẾN

http://www.scribd.co...oi-Ban-Cung-The

#6
alex_hoang

alex_hoang

    Thượng úy

  • Hiệp sỹ
  • 1152 Bài viết
Bài 6: (đề thi HSG QN 2010) cho 2 số thực dương x;y thoa $x+y+1=3xy$
tim max
$\dfrac{3x}{y(x + 1)} + \dfrac{3y}{{x.(y + 1)}} - (\dfrac{1}{{{x^2}}} + \dfrac{1}{{{y^2}}})\$

Bài viết đã được chỉnh sửa nội dung bởi dark templar: 28-05-2011 - 11:08
Latex

alex_hoang


HẸN NGÀY TRỞ LẠI VMF THÂN MẾN

http://www.scribd.co...oi-Ban-Cung-The

#7
dark templar

dark templar

    Kael-Invoker

  • Hiệp sỹ
  • 3788 Bài viết

Bài 6: (đề thi HSG QN 2010) cho 2 số thực dương x;y thoa $x+y+1=3xy$
tim max
$\dfrac{3x}{y(x + 1)} + \dfrac{3y}{{x.(y + 1)}} - (\dfrac{1}{{{x^2}}} + \dfrac{1}{{{y^2}}})\$

Xem ở đây
P/s:Bài 7:(IMO Shortlist) Cho $n$ số thực dương $a_1,a_2,...,a_n$ thỏa $ \sum\limits_{i=1}^{n}a_{i}<1$.Chứng minh rằng:
$\dfrac{a_1a_2...a_n[1-(a_1+a_2+...+a_n)]}{(a_1+a_2+...+a_n)(1-a_1)(1-a_2)...(1-a_n)} \le \dfrac{1}{n^{n+1}}$

Bài viết đã được chỉnh sửa nội dung bởi dark templar: 28-05-2011 - 11:15

"Do you still... believe in me ?" Sarah Kerrigan asked Jim Raynor - Starcraft II:Heart Of The Swarm.

#8
Nguyễn Hoàng Lâm

Nguyễn Hoàng Lâm

    Sĩ quan

  • Thành viên
  • 312 Bài viết

Bài 4 (chọn học sinh giỏi tỉnh Nghệ An 2010) cho các số thực dương a,b,c tỏa mãn $\[9({a^4} + {b^4} + {c^4}) - 25({a^2} + {b^2} + {c^2}) + 48 = 0\]$.Tìm GTNN của biểu thức
$\[\dfrac{{{a^2}}}{{b + 2c}} + \dfrac{{{b^2}}}{{c + 2c}} + \dfrac{{{c^2}}}{{a + 2b}}\]$

Bạn ơi check đề lại giùm mình được không , theo mình thì BĐT phải có dạng đối xứng chứ.
$\[\dfrac{{{a^2}}}{{b + 2c}} + \dfrac{{{b^2}}}{{c + 2c}} + \dfrac{{{c^2}}}{{a + 2b}}\]$ Cái đó là BCNN hả ?

Bài viết đã được chỉnh sửa nội dung bởi Nguyễn Hoàng Lâm: 28-05-2011 - 11:35

Đôi khi ta mất niềm tin để rồi lại tin vào điều đó một cách mạnh mẽ hơn .


#9
alex_hoang

alex_hoang

    Thượng úy

  • Hiệp sỹ
  • 1152 Bài viết

Bạn ơi check đề lại giùm mình được không , theo mình thì BĐT phải có dạng đối xứng chứ.
$\[\dfrac{{{a^2}}}{{b + 2c}} + \dfrac{{{b^2}}}{{c + 2a}} + \dfrac{{{c^2}}}{{a + 2b}}\]$ Cái đó là BCNN hả ?

Xin lỗi nha
alex_hoang


HẸN NGÀY TRỞ LẠI VMF THÂN MẾN

http://www.scribd.co...oi-Ban-Cung-The

#10
alex_hoang

alex_hoang

    Thượng úy

  • Hiệp sỹ
  • 1152 Bài viết
Bài 8:(IMO shortlist) cho các số thực dương a,b,c thỏa mãn $a + b + c = \dfrac{1}{a} + \dfrac{1}{b} + \dfrac{1}{c}$.CMR
$\dfrac{1}{{{{(2a + b + c)}^2}}} + \dfrac{1}{{{{(2b + c + a)}^2}}} + \dfrac{1}{{{{(2c + a + b)}^2}}} \le \dfrac{3}{{16}}$

Bài viết đã được chỉnh sửa nội dung bởi dark templar: 28-05-2011 - 17:06
Gõ bằng Mathtype rồi thì bỏ \[ ở đầu và cuối dòng

alex_hoang


HẸN NGÀY TRỞ LẠI VMF THÂN MẾN

http://www.scribd.co...oi-Ban-Cung-The

#11
truclamyentu

truclamyentu

    Sĩ quan

  • Thành viên
  • 333 Bài viết

Xem ở đây
P/s:Bài 7:(IMO Shortlist) Cho $n$ số thực dương $a_1,a_2,...,a_n$ thỏa $ \sum\limits_{i=1}^{n}a_{i}<1$.Chứng minh rằng:
$\dfrac{a_1a_2...a_n[1-(a_1+a_2+...+a_n)]}{(a_1+a_2+...+a_n)(1-a_1)(1-a_2)...(1-a_n)} \le \dfrac{1}{n^{n+1}}$



$\sum\limits_{i = 1}^n {{a_i}} < 1 \Rightarrow {a_1} + {a_2} + ... + {a_n} + {a_{n + 1}} = 1;{a_{n + 1}} > 0$


$ \Rightarrow \dfrac{{{a_1}{a_2}...{a_n}[1 - ({a_1} + {a_2} + ... + {a_n})]}}{{({a_1} + {a_2} + ... + {a_n})(1 - {a_1})(1 - {a_2})...(1 - {a_n})}}$


$ = \dfrac{{{a_1}{a_2}...{a_n}{a_{n + 1}}}}{{({a_1} + {a_2} + ... + {a_n})(\sum\limits_{i = 1;i \ne 1}^{n+1} {{a_i}} )(\sum\limits_{i = 1;i \ne 2}^{n+1} {{a_i}} )...(\sum\limits_{i = 1;i \ne n}^{n+1} {{a_i}} )}}$


$ \le \dfrac{{{a_1}{a_2}...{a_n}{a_{n + 1}}}}{{n\sqrt[n]{{{a_1}...{a_n}}}.{n^n}.\sqrt[n]{{\prod\limits_{i = 1;i \ne 1}^n {{a_i}} }}.\sqrt[n]{{\prod\limits_{i = 1;i \ne 2}^n {{a_i}} }}...\sqrt[n]{{\prod\limits_{i = 1;i \ne n}^n {{a_i}} }}}} = \dfrac{1}{{{n^{n + 1}}}}$

Bài viết đã được chỉnh sửa nội dung bởi truclamyentu: 28-05-2011 - 15:25


#12
dark templar

dark templar

    Kael-Invoker

  • Hiệp sỹ
  • 3788 Bài viết

Bài 8:(IMO shortlist) cho các số thực dương a,b,c thỏa mãn $a + b + c = \dfrac{1}{a} + \dfrac{1}{b} + \dfrac{1}{c}$.CMR
$\dfrac{1}{{{{(2a + b + c)}^2}}} + \dfrac{1}{{{{(2b + c + a)}^2}}} + \dfrac{1}{{{{(2c + a + b)}^2}}} \le \dfrac{3}{{16}}$

Từ giả thuyết,ta suy ra $ab+bc+ca \ge 3$
Áp dụng BĐT AM-GM,ta có:$(2c+a+b)^2 \ge 4(c+a)(c+b)$
Như vậy,ta có:$VT \le \dfrac{2(a+b+c)}{4(a+b)(b+c)(c+a)}$
Ta lại có 1 BĐT quen thuộc sau:$(a+b)(b+c)(c+a) \ge \dfrac{8}{9}(a+b+c)(ab+bc+ca)$
Nên ta có $VT \le \dfrac{9}{16(ab+bc+ca)} \le \dfrac{3}{16}=VP$
Vậy ta có đpcm.Đẳng thức xảy ra khi $a=b=c=1$
-------------------------------------------------------------------------------------------------------------------------------------------------------------------
P/s:Bài 9:
Cho $a,b,c>0;abc=1$.Chứng minh rằng:$\dfrac{a}{\sqrt{8c^3+1}}+\dfrac{b}{\sqrt{8a^3+1}}+\dfrac{c}{\sqrt{8b^3+1}} \ge 1$

Bài viết đã được chỉnh sửa nội dung bởi dark templar: 28-05-2011 - 17:59

"Do you still... believe in me ?" Sarah Kerrigan asked Jim Raynor - Starcraft II:Heart Of The Swarm.

#13
alex_hoang

alex_hoang

    Thượng úy

  • Hiệp sỹ
  • 1152 Bài viết
mục đích gủi BĐT bài 8 là dể thấy dc sư tương tự so với bài Việt Nam TST 2010
cho các số thực dương a,b,c thỏa mãn dk
$ \dfrac{1}{a} + \dfrac{1}{b} + \dfrac{1}{c} \le 16(a + b + c)$
CMR
$\dfrac{1}{[a + b + \sqrt {2(a + c)}]^3} + \dfrac{1}{[b + c + \sqrt {2(b + a)}]^3} + \dfrac{1}{[c + a + \sqrt {2(c + b)}]^3} \le \dfrac{8}{9}$

Bài viết đã được chỉnh sửa nội dung bởi dark templar: 28-05-2011 - 20:40
Không cần thêm \[ vào đầu và cuối mỗi cặp thẻ Latex đâu

alex_hoang


HẸN NGÀY TRỞ LẠI VMF THÂN MẾN

http://www.scribd.co...oi-Ban-Cung-The

#14
alex_hoang

alex_hoang

    Thượng úy

  • Hiệp sỹ
  • 1152 Bài viết

P/s:Bài 9:
Cho $a,b,c>0;abc=1$.Chứng minh rằng:$\dfrac{a}{\sqrt{8c^3+1}}+\dfrac{b}{\sqrt{8a^3+1}}+\dfrac{c}{\sqrt{8b^3+1}} \ge 1$

Bài 9 mình làm thế này
$8{c^3} + 1 = (2c + 1)(4{c^2} - 2c + 1) \le {\left( {\dfrac{{4{c^2} + 2}}{2}} \right)^2} = {(2{c^2} + 1)^2}$
$ \Rightarrow \dfrac{a}{{2{c^2} + 1}} \le \dfrac{a}{{\sqrt {8{c^3} + 1} }}$
ta chỉ cần cm
$\dfrac{a}{{2{c^2} + 1}} + \dfrac{b}{{2{a^2} + 1}} + \dfrac{c}{{2{b^2} + 1}} \ge 1$
từ đây qui đồng sd AM GM lưu ý abc=1 ta có đpcm
-------------------------------------------------------------------------------------------------------------------------------------------
@dark templar:Bạn Alex Hoang có post bài thì nhớ trình bày rõ ra nhé,không có nên post kiểu:quy đồng lên rồi sử dụng AM-GM.....Đó được xem là spam và mình sẽ xóa không thương tiếc.Thân.

Bài viết đã được chỉnh sửa nội dung bởi dark templar: 28-05-2011 - 20:44
Latex

alex_hoang


HẸN NGÀY TRỞ LẠI VMF THÂN MẾN

http://www.scribd.co...oi-Ban-Cung-The

#15
alex_hoang

alex_hoang

    Thượng úy

  • Hiệp sỹ
  • 1152 Bài viết
Bài 10: cmr vói các số thực dương $a,b,c,x,y,z$ ta có
$\dfrac{a}{{b + c}}(y + z) + \dfrac{b}{{c + a}}(z + x) + \dfrac{c}{{a + b}}(x + y) \ge 3\dfrac{{xy + yz + zx}}{{x + y + z}}$
(crux Mathematicorum)

Bài 11 CMR nếu $a,b,c,d \in [0,1]$ thì
$1 \le a+b+c+d+(1-a)(1-b)(1-c)(1-d)$
(Math Challenge)
alex_hoang


HẸN NGÀY TRỞ LẠI VMF THÂN MẾN

http://www.scribd.co...oi-Ban-Cung-The

#16
dark templar

dark templar

    Kael-Invoker

  • Hiệp sỹ
  • 3788 Bài viết

Bài 10: cmr vói các số thực dương $a,b,c,x,y,z$ ta có
$\dfrac{a}{{b + c}}(y + z) + \dfrac{b}{{c + a}}(z + x) + \dfrac{c}{{a + b}}(x + y) \ge 3\dfrac{{xy + yz + zx}}{{x + y + z}}$
(crux Mathematicorum)

Sử dụng BĐT cơ bản:$(x+y+z)^2 \ge 3(xy+yz+zx)$,ta đưa bài toán về việc chứng minh :
$\dfrac{a}{{b + c}}(y + z) + \dfrac{b}{{c + a}}(z + x) + \dfrac{c}{{a + b}}(x + y) \ge \sqrt{3(xy+yz+zx)}$
Sử dụng tính thuần nhất của BĐT,ta chuẩn hóa $x+y+z=1$
BĐT tương đương với:$\sum \dfrac{a}{b+c} \ge \dfrac{a}{b+c}x+\dfrac{b}{a+c}y+\dfrac{c}{a+b}z+\sqrt{3(xy+yz+zx)}(1)$
Theo BĐT Cauchy-Schwarzt,ta có:
$VP_{(1)} \le \sqrt{\sum \left(\dfrac{a}{b+c} \right)^2}\sqrt{\sum x^2}+\dfrac{\sqrt{3}\sqrt{\sum xy}}{2}+\dfrac{\sqrt{3}\sqrt{\sum xy}}{2}$
$\le \sqrt{\sum \left(\dfrac{a}{b+c} \right)^2 +\dfrac{3}{2}}\sqrt{\sum x^2 +2\sum xy}=\sqrt{\sum \left(\dfrac{a}{b+c} \right)^2 +\dfrac{3}{2}}$
Chúng ta sẽ chỉ việc chứng minh:$\sqrt{\sum \left(\dfrac{a}{b+c} \right)^2 +\dfrac{3}{2}} \le \sum \dfrac{a}{b+c}$
$ \Leftrightarrow \sum \dfrac{ab}{(c+a)(c+b)} \ge \dfrac{3}{4} \Leftrightarrow \sum ab(a+b) \ge 6abc $
$\Leftrightarrow a(b-c)^2+b(c-a)^2+c(a-b)^2 \ge 0$
(luôn đúng)
Vậy ta có đpcm.Đẳng thức xảy ra khi $ \left\{\begin{array}{l}a=b=c\\x=y=z\end{array}\right. $

Bài viết đã được chỉnh sửa nội dung bởi dark templar: 29-05-2011 - 08:32

"Do you still... believe in me ?" Sarah Kerrigan asked Jim Raynor - Starcraft II:Heart Of The Swarm.

#17
alex_hoang

alex_hoang

    Thượng úy

  • Hiệp sỹ
  • 1152 Bài viết
Bài 12(Math Camo 200) cho a,b,c là độ dài 3 cạnh của 1 tam giác nhọ CMR
$(a + b + c)({a^2} + {b^2} + {c^2})({a^3} + {b^3} + {c^3}) \ge 4({a^6} + {b^6} + {c^6})$

Bài 12(Math Camo 2000) cho a,b,c là độ dài 3 cạnh của 1 tam giác nhọ CMR
$(a + b + c)({a^2} + {b^2} + {c^2})({a^3} + {b^3} + {c^3}) \ge 4({a^6} + {b^6} + {c^6})$

Các bạn cũng post nhưng BĐT hay của mình lên đế cùng trao đổi nữa nha
alex_hoang


HẸN NGÀY TRỞ LẠI VMF THÂN MẾN

http://www.scribd.co...oi-Ban-Cung-The

#18
truclamyentu

truclamyentu

    Sĩ quan

  • Thành viên
  • 333 Bài viết

Bài 10: cmr vói các số thực dương $a,b,c,x,y,z$ ta có
$\dfrac{a}{{b + c}}(y + z) + \dfrac{b}{{c + a}}(z + x) + \dfrac{c}{{a + b}}(x + y) \ge 3\dfrac{{xy + yz + zx}}{{x + y + z}}$
(crux Mathematicorum)

Bài 11 CMR nếu $a,b,c,d \in [0,1]$ thì
$1 \le a+b+c+d+(1-a)(1-b)(1-c)(1-d)$
(Math Challenge)



Mình chém câu 11 nha : :Rightarrow :Rightarrow :Rightarrow

Đặt : $1-a=x ; 1-b =y ; 1-c=z ; 1-d=t$ suy ra $x,y,z,t \in [0;1]$ ,suy ra $xy ;zt \in [0;1]$ .

Khi đó viết lại YCBT : $xyzt \geq x+y+z+t-3$

Ta có : $(1-x)(1-y) \geq 0 \Rightarrow xy \geq x+y-1$

$(1-z)(1-t) \geq 0 \Rightarrow zt \geq z+t-1$

$(1-xy)(1-zt) \geq 0 \Rightarrow xyzt \geq xy+zt-1$

$\Rightarrow xyzt \geq xy+zt-1 \geq (x+y-1)+(z+t-1)-1 $
$\Rightarrow xyzt \geq x+y+z+t-3 \Rightarrow Q.E.D$

Dấu bằng xảy ra tại $a=b=c=d=0$ hoặc 1 số bằng 1 , 3 số còn lại bằng 0 .

Bài viết đã được chỉnh sửa nội dung bởi dark templar: 29-05-2011 - 11:07
Latex


#19
dark templar

dark templar

    Kael-Invoker

  • Hiệp sỹ
  • 3788 Bài viết

Bài 12(Math Camo 200) cho a,b,c là độ dài 3 cạnh của 1 tam giác nhọ CMR
$(a + b + c)({a^2} + {b^2} + {c^2})({a^3} + {b^3} + {c^3}) \ge 4({a^6} + {b^6} + {c^6})$
Các bạn cũng post nhưng BĐT hay của mình lên đế cùng trao đổi nữa nha

Bài 13:
Cho $a,b,c>0$.Chứng minh rằng:$\dfrac{a^3}{b^2-bc+c^2}+\dfrac{b^3}{c^2-ac+a^2}+\dfrac{c^2}{a^2-ab+b^2} \ge \dfrac{3(ab+bc+ca)}{a+b+c}$
Bài toán tổng quát: Cho $a,b,c>0;n \in N^*$.Chứng minh rằng:
$\dfrac{2a^{n}-b^n-c^n}{b^2-bc+c^2}+\dfrac{2b^n-c^n-a^n}{c^2-ca+a^2}+\dfrac{2c^n-a^n-b^n}{a^2-ab+b^2} \ge 0$
P/s:Bạn xem lại giùm đề Bài 12.
"Do you still... believe in me ?" Sarah Kerrigan asked Jim Raynor - Starcraft II:Heart Of The Swarm.

#20
alex_hoang

alex_hoang

    Thượng úy

  • Hiệp sỹ
  • 1152 Bài viết

Mình chém câu 11 nha : :Rightarrow :Rightarrow :Rightarrow

Đặt : $1-a=x ; 1-b =y ; 1-c=z ; 1-d=t$ suy ra $x,y,z,t \in [0;1]$ ,suy ra $xy ;zt \in [0;1]$ .

Khi đó viết lại YCBT : $xyzt \geq x+y+z+t-3$

Ta có : $(1-x)(1-y) \geq 0 \Rightarrow xy \geq x+y-1$

$(1-z)(1-t) \geq 0 \Rightarrow zt \geq z+t-1$

$(1-xy)(1-zt) \geq 0 \Rightarrow xyzt \geq xy+zt-1$

$\Rightarrow xyzt \geq xy+zt-1 \geq (x+y-1)+(z+t-1)-1 $
$\Rightarrow xyzt \geq x+y+z+t-3 \Rightarrow Q.E.D$

Dấu bằng xảy ra tại $a=b=c=d=0$ hoặc 1 số bằng 1 , 3 số còn lại bằng 0 .

cau 11 này mình đưa lên là muốn khởi đầu cho 1 pp mới pp cm BĐT dựa vào hàm số bậc nhất
lời giải đó như sau
không mất tính tổng quát ta coi biểu thức bên vế phải là một hàm bậc nhất với ẩn a
Giá trị của f(a) đạt dược tại 1 trong 2 diem mút của đoạn 0,1
nếu a=1 thì f(1)=1+a+b+c+d>1
nếu a=0 thì
f(0)=b+c+d+(1-b)(1-c)(1-d)
lại coi f(0) là hàm bậc nhất g(b)
lặp lại qt trên cuối cùng nếu a=b=c=d=0 thì vế phải =1
Một số bài toán rèn luyện pp (cái này không nằm trong topic đâu)
cho x,y,z là các số thực dương thỏa mãn x+y+z=1
CMR
a, $5({x^2} + {y^2} + {z^2}) \le 6({x^3} + {y^3} + {z^3}) + 1$
b, ${x^3} + {y^3} + {z^3} + 6xyz \ge \dfrac{1}{4}$
câu a ở old and new vasile citoaje
câu b USAMO 1979
alex_hoang


HẸN NGÀY TRỞ LẠI VMF THÂN MẾN

http://www.scribd.co...oi-Ban-Cung-The




0 người đang xem chủ đề

0 thành viên, 0 khách, 0 thành viên ẩn danh